« first day (30 days earlier)      last day (544 days later) » 
00:00 - 15:0015:00 - 00:00

12:04 AM
0
Q: Find the sum of the integers in the continued fraction

user19405892Find the sum of integers $a,b,c,d,$ and $e$ if $\dfrac{2011}{1990} = a+\dfrac{1}{b+\dfrac{1}{c+\dfrac{1}{d+\dfrac{1}{e}}}}$.

Questions tend to get more attention when they have a tag for a broad area of mathematics relevant to the question. Some of these tags might fit. (from a bot)Normal Human 21 secs ago
0
Q: Find the sum of the integers in the continued fraction

user19405892Find the sum of integers $a,b,c,d,$ and $e$ if $\dfrac{2011}{1990} = a+\dfrac{1}{b+\dfrac{1}{c+\dfrac{1}{d+\dfrac{1}{e}}}}$. I could simply the big fraction on the RHS, but I don't see how that would help. Also, there are infinitely many solutions to this equation so how should I find the intege...

Questions tend to get more attention when they have a tag for a broad area of mathematics relevant to the question. Some of these tags might fit. (from a bot)Normal Human 21 secs ago
0
Q: Reduction in uncertainty

laura herrmann" I(X;Y|Z) is interpreted as `` the reduction in uncertainty of X due to the knowledge of Y when Z is given." Would it make sense when talking in a geographical sense to say : When information flow in the EU (Z) occurs the flow of interaction between the countries (I) increases due to labour f...

0
Q: Prove the following equation has a unique solution: x^101 + x^99 + 5x + 2 = 0

fruity.shine.chapstickProve the following equation has a unique solution: x^101 + x^99 + 5x + 2 = 0 (Use continuity for existence and Roll Theorem for uniqueness) I have no idea where to start and how to prove.

Welcome to Math.SE, fruity.shine.chapstick. This site uses MathJax formatting of formulas. More tips here. (from a bot)Normal Human 21 secs ago
 
0
Q: Strange "Ghost" +1 showing up in my reputation changes

KrythicI was just wondering if anyone else has been encountering a strange "ghost" +1 reputation change on the stackexchange header. I have been seeing this frequently, perhaps once every two to three days. Here is a picture: If I click on it, there will be nothing gained though. I have no updates sh...

 
12:30 AM
0
Q: Stochastic Calculus Proof

Dr. Ikjyot Singh KohliI'm working on the following problem in the realm of stochastic calculus, and don't really know how to proceed. I have a stochastic process, call it $X(t)$. I have a function defined as: $V = 3H^2 X^2$, where $H$ is some non-random real variable. The question is I have to show that for: $...

 
12:43 AM
0
Q: Convolution is smooth

shilovI would like to show the convolution of two functions is smooth. $\newcommand{\Rn}{\mathbb{R}^N}$ $\newcommand{\dxi}[2]{\frac{\partial #1}{\partial x_{#2 } } } $ $\newcommand{\conv}[2]{#1 \star #2}$ $\newcommand{\ball}{B(0, {1 \over n})}$ $\phi_n \in C^\infty_c( \Rn )$ which is a mollifier. $f\...

 
12:57 AM
0
Q: Why is this a fake proof?

ra1nmasterI am aware of the "definition" of the total differential as follows: $$\mathrm{d}f = \frac{\partial f}{\partial x} \mathrm{d}x + \frac{\partial f}{\partial y} \mathrm{d} y.$$ Now, assume we wished to show that: $$\frac{\mathrm{d}y}{\mathrm{d}x} = - \frac{\partial f / \partial x}{\partial f / \...

0
Q: Assume that $S$={(x,y): x-y=0}. Find the projection formula $P_S$.

whatarethoseQ: Assume that $S$={(x,y): x-y=0}. Find the projection formula $P_S$. I'm a bit stuck on where to even begin? Any help would be great!

0
Q: I think it's sultan's daughter problem, but prf' says it aint help me

기세진'To select optimal floor.' is the quiz. Suppose you have 10th floor flat, and you have only the elevator that can asecending not go down. You have girls in every floor (1 per each floors). Which way I can select the floor that optimal in probability? I simply can do the answer because prf say...

0
Q: Power series of Isometries

STCJI was asked to show, given an isometry U from a finite dimensional inner product space V to itself that $$ A_n(x)=\frac{1}{n}\sum_{0}^{n}U^{n}(x) \to 0 \text{ as } n\to \infty \text{ for} x\in Im(I-U) $$ so letting $ y-U(y)$ $$ A_n(x)=\frac{1}{n}\sum_{0}^{n}U^{n}(y-U(y))=\frac{1}{n}\sum_{0}^{n}U...

0
Q: An exercise concerning infinite products

pozioI am stuck with the following exercise: let $\alpha$ be a complex number which is not integer. Prove $$\frac{\sin\pi(z+\alpha)}{\sin\pi\alpha}=e^{\pi z\cot\pi\alpha}\prod_{n=-\infty}^\infty \left(1-\frac{1}{n-\alpha}\right)e^{\frac{z}{n-\alpha}}$$ I tried to prove this in the following way: us...

0
Q: sum of sigma finite measure

Mario PérezI want to demonstrate: Finite sum of sigma-finite measures is a sigma finite measure. I try this: Suppose that are m sigma measures in a space ($\mathcal{X}$,$\mathcal{A}$), denoted by $\mu_k$ with $k \in {1,...,m} $. Then exist $A_n^k$ for each k,and $\mathcal{X}=\bigcup\limits_{n \in \mathbb{...

0
Q: rotate complex eigenvector

Bowen ZhaoI'm computing the eigenvectors of a real non-symmetric Matrix. I know that complex eigenvectors would come in conjugate. However, does anybody know a algorithm to rotate such complex eigenvectors that i) its real and imaginary parts are orthogonal and ii) real part has norm 1 while imaginary par...

0
Q: When except for gcd$(0,0)$ does a greatest common divisor equal $0$?

JimmWhen except for gcd$(0,0)$ does a greatest common divisor equal $0$? gcd$(x,0)$, for example, = $x$, not $0$.

0
Q: Name of this operation?

Anton BelovGiven two tuples $A = \langle a_1, \dots, a_n \rangle$ and $B = \langle b_1, \dots, b_m \rangle$, let $A\oplus B \stackrel{def}{=} \langle (a_1+b_1), \dots, (a_1+b_m), (a_2+b_1), \dots, (a_2+b_m), \dots, (a_n+b_m) \rangle$, where $+$ is just the addition (e.g. over $N$). Is there a known name for...

0
Q: Given a biased die, what some of the probabilities

BNSlugI recently asked this question: Roll a fair 6 sided die twice, What is the probability that one or both rolls are 6? And that makes sense, however, the reason I was confused in the first place is because we went over the following question during section: Suppose the six-sided die you used in t...

This site uses MathJax formatting of formulas. More tips here. (autocomment)Normal Human 21 secs ago
 
1:35 AM
8
Q: Request for community attention on a moderator's behavior

NoobleIt has come to a handful of people's attention that at least some of the newly elected moderators might not be exactly up to challenge. Here's a message that very clearly demonstrates this incompetence, that was a result of a particular argument that appeared in one of the SO chatrooms: and a...

 
1:53 AM
0
Q: Curves Knotted in the Torus

Alfred YergerI've been struggling with the following problem. I suspect it's actually not so hard, but I'm missing something relatively obvious about the proof. The attached image will help. Suppose $K$ and $L$ are two curves inside the solid torus $\mathbb S^1 \times D$, which are each on one side of the ho...

 
-1
Q: Navigating 'Back' from question changes my custom tab settings

AlexanderIt's similar to Can't go forward after going back to question list when using custom tabs where I have a customized tab like this: But when I open a question, decide that it's not interesting and click on the back button of chrome, the tab looks like this: The filter is set to all and order b...

 
0
Q: How to prove p(x +a) = p(x) + a

Jair ReyesAs far as I know if you have p(x) = x ^ a + x ^ b + x ^ c + .... + x ^ d; and then you want to know p(x + Z). p(x + Z) = x ^ a + x ^ b + x ^ c + .... + x ^ d + Z; IN other words p(x + Z) = p(x) + Z. how to prove it?

Welcome to Math.SE, Jair Reyes. This site uses MathJax formatting of formulas. More tips here. (autocomment)Normal Human 21 secs ago
0
Q: Show that the function $f(x) = \begin{cases} \frac{x^2y^4}{x^4+y^8} ,& \text{if } (x,y)≠ (0,0) \\ 0, &\text{if } (x,y)= (0,0)\end{cases}$

user291528Show that the function $$f(x) = \begin{cases} \frac{x^2y^4}{x^4+y^8} ,& \text{if } (x,y)≠ (0,0) \\ 0, &\text{if } (x,y)= (0,0)\end{cases}$$ is Gateaux differentiable at $(0,0)$ but not continuous at $(0,0)$. Not too sure how to start this. I know there is the Gateaux differentiability forumula...

0
Q: Set Theory (Counting Sets)

bestbowlerWhen Amy and John go out for dinner it is either just the two of them, or the two of them together with one or both of their two closest friends. This month they have gone out with each of these friends a total of 7 times, and with both of them together 3 times. If, over the month, Amy and John h...

0
Q: Geometric Sequences in Manufacturing

user93096I came up with this problem: The manufacturer of a company plans to produce and sell 8000 units per year. Each year, 10% of the units become inoperative. So basically, I need a definition for this series, recursive or explicit it does not matter. I know that A1=8000 + .9(8000), A2=8000 + .9(A1)....

Question contains please. Geometric Sequences in Manufacturing
 
2:21 AM
0
Q: Calculating the derivative with limited info.

Sw JamesG(x) := integral(f(t)) dt from x to x^2 Calculate G'(x). I've made some progress by integrating by parts with f(t) = 1(f(t)) but I'm stuck now and don't know where to go.

0
Q: Modulus of complex number less then equal to 1

DeliaIt $|z+1|\le 1 \text{and} |z^2+1|\le 1$, then we have $$ |z|\le 1.$$ I wrote $z=x+iy, x,y\in \mathbb{R}$ and the inequalities from hypothesis become \begin{equation} (x+1)^2+y^2\le 1 \text{ and } (x^2-y^2+1)^2+4x^2y^2\le 1 \end{equation}...and I don't see how to deduce from here $$x^2+y^2\le 1.$...

0
Q: Rational Power Question

LukeShow that if a ∈ Q is positive and if 0 < x < y then x^a < y^a. I was told to use the difference theorem for this question, but the difference theorem is only for natural numbers.

Words such as question are uninformative in titles. Please edit the title so that it better describes the specifics of your question. Do not hesitate to make it longer or include a formula if needed. This site uses MathJax formatting of formulas. Tag (proof-writing) should not be the only tag a question has. Please add a tag for a subject area to which the question belongs. More tips here. (autocomment)Normal Human 21 secs ago
 
2:45 AM
0
Q: Linear combination of group elements as a ring?

user139985 I'm having trouble understanding this example. Why is $\mathbb{Z}$G considered a ring? How would this be shown? Any advice would be helpful.

0
Q: Is the following proof correct?

JimmIs the following proof correct? Let’s say we find integers $x$ and $y$ such that $x^2 ≡ y^2($mod $n)$ and $n$ has at least 2 distinct factors not equal to 0 or n. I intend to show that there is at least $50$% chance that gcd$(n, x - y)$ is a non-trivial factor of $n$. (Assume already proven tha...

Tagged proof-verification. Is the following proof correct?
 
0
Q: How to type special characters in this forum?

SolitaryThe general rule used in LaTeX doesn't work: for example, typing M\"{o}bius and Cram\'{e}r doesn't give the desired outputs.

-1
Q: javascript dynamic get different element height in safari and chrome browser

AnamiI have a water-fall layout in container area, when it show on chrome was perfect but in safari, all element are stack up above another ones. I found a problem seems like i can't quite actually get real element height in safari by javascript. attached my code JS: for(var i=0, l=flows.lengt...

 
3:05 AM
0
Q: clarification of inequalities

tmnetI'm reading this proof (page 7), and get confused by the ending part , quoted here: Take $c$ a constant less than $\sqrt{2}$. Then Since $var(x) \leq n^{3-2c}+n^{2-c^{2}}, E(x) \leq n^{2-c^{2}}$, and $E^{2}(x) \leq n^{4-2c^{2}}$, it follows that $var(x) \leq E^{2}(x)$. What's the reason ...

0
Q: Throwing a die until an uneven number occurs

Metanoia.RichiIn probability, an experiment consisting of throwing a die until an uneven number occurs. How do I solve this ? Please Help T^T

Short question. Question contains please. Throwing a die until an uneven number occurs
0
Q: Establish the convergence or divergence of the series whose nth term is $\frac{n}{(n+1)(n+2)}$

user290591The series whose nth term is $\frac{n}{(n+1)(n+2)}$, I have to check the convergence or divergence of it. So $$a_n=\frac{n}{(n+1)(n+2)}=\frac{2}{n+2}-\frac{1}{n+1}$$ Hence as $n\to \infty$, $a_n\to 0$ so the series is convergent. Am I correct?

0
Q: Proof by Induction $\sum_{i=0}^{k-1} 2^i = 2^n-1$

Juan SalazarProve mathematical induction $\sum_{i=0}^{k-1} 2^i = 2^n-1$

 
3:23 AM
0
Q: Does 'pay some effort' make a question about negligible performance valid?

GstestsoI just see this question about performance of a line of code, it is quite well-receiving now, but unfortunately this type of questions may receive negative feedback like this. To be my honest, I may learn a lot new skills from these type of questions even the original performance problem is not ...

0
Q: How can the following possibly fail a grammar test?

Bill DrisselQuestion> I'm bringing up a new installation of MinGW 4.8.1. gcc and g++ commands eventually call cc1.exe. cc1 fails to even execute and dies with "[long path]/cc1.exe: error while loading shared libraries: ?: cannot open shared object file: No such file or directory". I'm baffled by the "?"....

 
0
Q: Is it $(0,1)$ an compact set?

tankonetooneIt is well known that open interval $(0,1)$ is not compact on real line. But I somehow remembered that if we take the open interval $(0,1)$ itself as a metric space, then it is a compact metric space. Is this true? Sorry if this question is naive...

0
Q: Hamilton Circuit, prove

soullessIn graph G, there is a Hamilton Path from vertex v to vertex m, with v and m not adjacent. Suppose that G is a simple graph with n vertices, n >= 3, and deg(x) + deg(y) >= n whenever x and y are nonadjacent vertices in G. To prove or disprove that G has a Hamilton Circuit. My teacher gave us an ...

0
Q: Confused on how to find the interior of a set

whatarethoseI have to show whether or not the set $[0,1] ∪ [2,3]$ has an interior of (0,1). I'm still really confused how to determine the interior of a set. Can someone please explain?

Title contains confus. Short question. Question contains please. Confused on how to find the interior of a set
0
Q: Verify the Product of a Summation

NicCan anybody verify that the below equation equals $0$? $\prod\limits_{k=2}^{10} (\sum\limits_{i=1}^{k-1}(2(i-1)))$ Here is my work, I believe it's correct:

0
Q: Prove that if $b \in aH$, then $aH = bH$.

O. SmithLet $H$ be a subgroup of $G$. Let $a$, $b \in G$. Prove that if $b \in aH$, then $aH = bH$.

 
4:01 AM
0
Q: Integration involving bessels function

ANKIT GUPTAI have to integrate the following : $$\int_0^\infty x^{s-1}e^{(-ax^p-bx^{-q})}$$ Please help me I am not able to solve this .. However there is this identity available in a book $$\int_0^\infty x^{s-1}e^{(-ax^p-bx^{-p})} = 2 p^{-1}(\frac{b}{a})^{(\frac{h}{2s})}K_{(\frac{s}{h})}(2a^{(\frac{1}{...

Question contains please. Integration involving bessels function
 
4:22 AM
0
Q: $\alpha (1+\alpha/2)^{-1} > \log(1+\alpha) $

user276387How does one prove that $\alpha (1+\alpha/2)^{-1} > \log(1+\alpha) $ for $\alpha > 0$?

A title should not be all-MathJax; having some plain text helps with search and navigation. (autocomment)Normal Human 21 secs ago
0
Q: $T:f(x)\to f(x-1)+x^3f'''(x)/3$ Find the Jordan normal form and a Jordan basis for $T$.

kileLet $T\in \mathcal{L}(\mathcal{P_3}(\mathbb{C})$ be the operator $$T:f(x)\to f(x-1)+x^3f'''(x)/3$$ Find the Jordan normal form and a Jordan basis for $T$.

0
Q: Laplace Transform of equation

AlexI'm having trouble with the laplace transform: $\mathcal{L} \lbrace \sqrt{\frac{t}{\pi}}\cos(2 t) \rbrace$ The problem gives me the transform identity $\mathcal{L} \lbrace \frac{\cos(2 t)}{\sqrt{\pi t}} \rbrace = \frac{e^{-2/s}}{\sqrt{s}}$ but i'm not sure/confused as to why that would help me

0
Q: Singular Perturbation Approx. for $\epsilon y'' + \frac{2 \epsilon}{t} y'-y=0$

MC989Use singular perturbation techniques to fi nd the leading order uniform ap- proximation to the solution to the boundary value problem $$\epsilon y'' + \frac{2 \epsilon}{t} y'-y=0$$ $0<t<1$ and $y(0)=0 , y'(1)=1$ This has a boundary layer near $t=1$ I am having trouble figuring out how to co...

Questions tend to get more attention when they have a tag for a broad area of mathematics relevant to the question. Some of these tags might fit. (autocomment)Normal Human 21 secs ago
0
Q: Prove a metric inequality

lawrenceli9308For points x , y in $\mathbb{R}^k$, let $$d_1(\boldsymbol{x},\boldsymbol{y}) = max\left \{ \left | x_j - y_j \right | : j = 1,2,...,k\right \}$$ $$d_2(\boldsymbol{x},\boldsymbol{y}) = \sum_{j = 1}^{k}\left | x_j - y_j \right |$$ Let $\vec{x}$ , $\vec{y}$ $\in$ $\mathbb{R}^n$. prove that for a...

 
4:55 AM
-1
Q: The Canonical navigation bar has diverged from the canon

Normal HumanPossibly due to the recent CSS refactoring, the pseudo-Canonical navigation bar no longer matches the top bar of ubuntu.com sites. Here is how they compare in my Chrome (Windows): AU bar Canonical bar Possible cause: although both sites declare Ubuntu as the font for the top bar, only ubun...

 
0
Q: Prove that for all n>11 we can represent $n$ by $n=3a+7b$

SorfoshSo i decided to do this using normal induction. P(12) true since 12=4*3 $P(k)=3a+7b$ $P(k+1)=3(a-2)+7(b+1)$ So i think it is proven but i cannot see why we have to assume n is larger than 11. I mean i know it is impossible for 11 but lets say 9. P(9) true since 9=3*3 $P(k)=3a+7b$ $P(k+1)=3(a-2)+7...

 
5:14 AM
0
Q: Do links to external images need editing or commenting?

Saschafrom time to time I see posts, that link to exteranl images and I want to know if and how I should comment the posts. I found this related question, but I am still unsure. The post that got me asking this question is this one. The image linked to the question is a screenshot of the error dialog ...

0
Q: Should I downvote answers without explanation?

SweeperI see this post: Get a specific value from comma separated string And there are two answers which is just an answer and does not provide any explanation. The OP just says that "use this" or "do that" and shows the code. I don't think these answers are in good quality. If I were the asker and saw...

 
0
Q: An exponential family problem

Lysla RiI don't know how to express the problem to the form $f(x|\theta)=h(x)c(\theta)(\sum_{i=1}^{k}\omega_i(\theta)t_i(x))$ Let $X$ have pdf $f(x)=\frac{1}{\beta}e^{-(x-\alpha)/\beta}$, $x>\alpha$ Determine whether $f(x)$ is an exponential a. if both $\alpha$ and $\beta$ are unknown. b. if only $\b...

Short title. Title contains problem. An exponential family problem
 
5:31 AM
0
Q: Show that a matrix has strictly positive real eigenvalues topology exercise

grayQuantLet $M$ be a $3 \times 3$ matrix with strictly positive real entries. Let $S \ = \ \{ \left. \ \left(\begin{array}{c} x \\ y \\ z\end{array}\right) \in \mathbb{R}^3 \ \right| \ x \geq 0, \ y \geq 0, \ z\geq 0, \ x^2+y^2+z^2=1 \}.$ By considering the map $f:S \to S$ defined by $\mathbf{x} \...

0
Q: Quotation ring \mathbb{Z}/4\mathbb{Z}

flipbackI'm trying to understand what the quotation ring is. I know that $\mathbb{Z}/4\mathbb{Z} = \mathbb{Z}_4$, but I can't get the same result by myself. Having used the definition of the quotation ring that $R/I = \left \{x + I | x \in R \right \}$ I've got: $$ 4\mathbb{Z} = \left \{4x | x \in \math...

0
Q: Closed subset of reals has continuous function vanishing on it

EvanSuppose $A \subset \mathbb{R}$ is closed. How can we show that there exists a continuous real-valued function $f$ such that $ker(f)=A$?

0
Q: Factorial Series as Recursive

NicI have a factorial series as shown below: \begin{equation} (2n+1)!~\text{for all $n \geq 0$} \end{equation} And I would like to know if the recursive definition that I wrote is accurate: \begin{equation} Factorial(n)=\begin{cases} n, & \text{if $n<0$}.\\ (2n+1)\cdot Factorial((2n+1)-...

0
Q: Binary vector which has the minimum numbers of 1

Yuriy PeysakhovAssume we have $p + 1$ vectors $a_1, a_2, \ldots ,a_p, b \in \left\{0,1\right\}^n$. How to find $\alpha_1, \alpha_2, \ldots ,\alpha_p\in\left\{0,1\right\}$ that vector $y$: $$y =\sum_{i=1}^p \alpha_i a_i + b$$ has the minimum numbers of $1$ ? All operations are on based $2$.

 
5:46 AM
0
Q: Question deleted as abandoned although I didn't abandon it

fifaltraI asked this question a while ago. It didn't receive any helpful attention, not even a comment asking for output of some command or something, or suggesting what I could look at. Since I still have the problem that is described there, I am still waiting for some reaction, although I do realise t...

0
Q: Feature request: &page=last link for paginated results

tripleeeMy Stack Overflow profile contains a link to the lowest-voted answers of mine; but it currently requires updating every 20 answers or so because I had to hard-code the index of the last page of results (currently at 84). Since the UI already knows that there are 84 pages (there is already a link...

 
0
Q: MATLAB code for mean radial distance of shape in a binary image

Muhammad ShoaibI want to create my own function for labeling and regional properties of binary in MATLAB. Please help in coding mean radial distance of shapes in binary images.

Questions tend to get more attention when they have a tag for a broad area of mathematics relevant to the question. Some of these tags might fit. (from a bot)Normal Human 21 secs ago
 
5:58 AM
0
Q: Hahn-Banach theoren(normed space)

NazoLet f be a bounded linear functional on a subspace Z of a separable normed space X. Theb there exist a bounded linear functional l on X which is an extension of f to X and has same norm.

Short title. Short question. Hahn-Banach theoren(normed space)
 
0
Q: Fake activity mark on favorites tab

მამუკა ჯიბლაძეI already asked about activity on favorites (How to find out what activity activated a favorite?). This time it is about what seems to be a bug. Every now and then the favorites tab shows new activity there. When I open it and sort by activity, the newest ones are actually few days ago and I defi...

 
0
Q: Find a parametric representation for an implicit function f(x,y,z)=0

HighmanI encountered a very interesting implicit function: $ z \exp \left[ (x-0.5-e^{z-y})^2+y^2-0.2z+3 \right] = \sin \left[ (xz-0.5)^2+2xy^2-0.1z \right]$ I wonder if there is any ways to find a parametric representation for it.

0
Q: LOGIC Question.Please help me out

Rider790A flight starts from Bangalore at 7 am local time and reaches Dubai at 12:30 am local time. Another flight starts from Dubai at 3 pm local time and reaches Bangalore at 8:40 pm local time. Both flights travel at a constant speed of 800 km/hr. Find the distance between Bangalore and Dubai.

Words such as question, please, help are uninformative in titles. Please edit the title so that it better describes the specifics of your question. Do not hesitate to make it longer or include a formula if needed. More tips here. (from a bot)Normal Human 21 secs ago
0
Q: What is the probability that a license plate under the proposed system will contain no duplicate letters or numbers? - Permutations

Aviral GuptaSo I saw this question. Office wants to create a new car licensing system in which each license plate will have seven characters: three letters (from the 26 letter alphabet) followed by four digits What is the probability that a license plate under the proposed system will contain no duplicate ...

This site uses MathJax formatting of formulas. More tips here. (from a bot)Normal Human 21 secs ago
0
Q: Multivariable calculus question about tangent planes.

user3867875I have a problem set due soon and I have been trying to fix my questions but I am stuck on one question in particular: "Are there any points on surface $x^2 - y^2 - z^2 = 1$ where the tangent plane is parallel to the plane $z=x+y$? [Hint: Remember that the normal vector to the tangent plane to a...

Words such as question are uninformative in titles. Please edit the title so that it better describes the specifics of your question. Do not hesitate to make it longer or include a formula if needed. More tips here. (autocomment)Normal Human 21 secs ago
0
Q: Notation confusion for joint pdf

shoestringfriesI have a pdf for continuous random variables X and Y such that $f_{xy}(x,y) = 2(x+y)$ for all $0<x<y<1$ and 0 elsewhere Does this mean for $0<x<1$ and $ 0<y<1$ or $0<x<y$ and $x<y<1$? I have to find $E[X^2Y]$

Title contains confus. Notation confusion for joint pdf
0
Q: laplace transform on differential equation system

kostasenter image description here I'm trying to solve the problem of the above picture.Can anyone help.

0
Q: Sufficient for homeomrophism: Sends basic open sets to basic open sets?

AlanI'm working on Atiyah-Macdonaldand in problem 26's Introduction to Commutative Algebra, and in chapter 1, problem 26, the goal is to show that for a compact Hausdorff space X, the space is homeomorphic to the subspace of Spec(C(X)), the zarinski topology on the continuous funtions of X, that ...

0
Q: Recommendations on visualizing basic linear algebra

KotelKanimI am teaching linear algebra this semester, and I would really like to recommend my students some cool youtube videos visualizing some simple stuff like the span of a set of vectors, linear dependence, subspaces (and their intersection and sum) and how does a basis give you a "coordinate system"....

0
Q: What's wrong with this integral calculation?

feralinI want to calculate the integral $$I = \int_0^{2 \pi} \cos^2 \theta\ \sin^4 \theta\ d \theta$$ by converting it into a complex integral around the unit circle. I use the identities $$\cos \theta = {1 \over 2} (e^{i \theta} + e^{-i \theta}),$$ $$\sin \theta = {1 \over 2 i} (e^{i \theta} - e^{-i ...

0
Q: deriving the function from a table of x-y values

feljgragmaIs there already a established rule on how to get the function (say cubic function) given a table of x-y values?

 
7:03 AM
0
Q: A canonical relativistic rocket question?

John RennieJohn Baez's old web site has disappeared, presumably since he's no longer associated with the university whose server they were on. In particular his page on the relativistic rocket equations is gone, and that was a useful article for gathering all the equations together. I was thinking of askin...

1
Q: What should I do with an answer, which brings nothing new compared to already given ones?

Andrey DeinekoI lately often see answers which repeats or just brings the same idea as the one(s) that have already been posted. I deem it as a simple chase of points, as if like better wording or something would attract the OP attention. Recent example: Question Answer given 4 hours ago (from now) Answer...

 
0
Q: Is there a monotonically increasing sequence that is bounded?

GniruTI just read the following theorem: If a sequence of real numbers is increasing and bounded above, then its supremum is the limit. How is it possible to have an increasing but bounded above sequence? Can you give me an example please?

0
Q: A question regarding independeance

whoisitSorry for this simple question, I'm a first year student, it's really basic but I don't know how to answer The question is: A random experiment is conducted which has sample space Ω {1,2,3,4,5,6,7,8,9,10,11,12,13,14,15,16,17,18,19,20} . Assume that all elementary events are equally likely to occu...

Welcome to Math.SE, whoisit. Words such as question do not add information to titles. Please edit the title so that it better describes the specifics of your question. Do not hesitate to make it longer or include a formula if needed. More tips here. (autocomment)Normal Human 21 secs ago
0
Q: max{U1(0,1),U2(0,1)} fits what distribution?

周庆特U1 and U2 are IID U(0,1). I want to know why max{U1,U2} ~ RT(0,1) The cdf of RT(0,1) is as follows. enter image description here

0
Q: Determine the sign of an integral without evaluating it

Refnom95This is from a real analysis module. I'm supposed to determine whether this integral is positive or negative without evaluating it. I have no idea how. Obviously x*cos(x) is positive over (0,pi/2) and negative over (pi/2,pi), so whichever partial integral is larger obviously determines the sign o...

This site uses MathJax formatting of formulas. More tips here. (from a bot)Normal Human 21 secs ago
0
Q: Determining poles and order of 1/sin(z)

IllustionistHow can I find the poles and the order of $1/sin(z)$?

0
Q: Fourier transform of a non linear diffusion

OhmWhat would be the Fourier transform of $\nabla^2 f(x)^2 $ ? (Where $y$ is some function of a space variable $x$)

 
7:43 AM
0
Q: How to use composition to generate random variates?

周庆特I want to know how to use composition to generate random variates whose cdf and pdf are as follows. CDF and PDF of the aimed distribution

0
Q: What is the contradiction here…? (Algebra)

Hawk Why is that a contradiction? I thought we assumed $a \neq 0$, so what are they talking about?

Short question. Tagged proof-explanation. What is the contradiction here…? (Algebra)
 
-3
Q: How to get my domain active?

Eugene HanI am having an issue with activating a domain. The domain is www.vidgifly.com and when you hit http://www.vidgifly.com/ you should see: It looks like a dead or expired domain :( I have installed apache2, php, mysql as well as virtualhost. Other virtualhosts are working perfectly but this one i...

 
0
Q: I am calculating the Shannon entropy and stuck on this integral

TBBTI am calculating the Shannon entropy of $\left|\Psi_{+}\left(x_{+}\right)\right|^{2}=\frac{1}{\sigma^{3}_{+}\sqrt{2\pi}}x^{2}_{+}\exp\left\{-\frac{x^{2}_{+}}{2\sigma^{2}_{+}}\right\}$, which is given by $$ H=-\int\left|\Psi_{+}\left(x_{+}\right)\right|^{2}\ln\left|\Psi_{+}\left(x_{+}\right)\right...

0
Q: Graph thoery tree rank question

user152711let $T=(V,E)$ be a tree with $n$ vertices , which his degree sequence is $d_1 \le d_2 \le ... \le d_n$ if and only if $d_1\gt 0$ and $d_1+d_2+...+d_n=2(n-1)$ $$$$ i managed to prove it one direction with indaction... but for days im not able to prove its a tree...(in the other direction). Please ...

Words such as question are uninformative in titles. Please edit the title so that it better describes the specifics of your question. Do not hesitate to make it longer or include a formula if needed. More tips here. (autocomment)Normal Human 21 secs ago
0
Q: Prove that $n$ is prime

VikramEstablish the following test for primes. If $n$ is odd, greater than $5$, and there exist relatively prime integers $a$ and $b$ such that $a — b = n$ and $a + b = p_1\cdot p_2\cdot... p_k$ (where $p_1, p_2 , . . . , p_k$ are the odd primes less than $\sqrt n$ ), then $n$ is prime.

0
Q: Density of real numbers

Mi-lee WilsonHow do I go about a proof to show that both the set of rational numbers and the set of irrational numbers are dense in R(real number).

Short title. Short question. Density of real numbers
 
8:03 AM
0
Q: Proving that a transformation is Unitary

byteSlayerLet $T:V\rightarrow V$ be a linear, normal transformation (meaning $TT^*=T^*T$) where it is known that $T^{-1} = -T$. Can it be proved that $T$ is unitary (e.g. $TT^*=T^*T=I$)?

0
Q: Three variable calculation

FarrukhDavid's truck can load 28tons and 70m³ of cargo and it costs $19600 to move from point A to point B. Mike wants to send his washing machine from point A to point B by David's truck. Washing machine's weight is 300kg, and it's volume is 1m³. How much should David charge Mike? Cost for every kg= ...

0
Q: Finding the least-squares solution of Ax = b when the columns of A are orthonormal.

bhaFind a formula for the least-squares solution of Ax = b when the columns of A are orthonormal.

Welcome to Math.SE, bha. Questions tend to get more attention when they have a tag for a broad area of mathematics relevant to the question. Some of these tags might fit. (from a bot)Normal Human 21 secs ago
 
8:19 AM
0
Q: Prove that $A_n$ is normal in $S_n$

Melissa HerringtonHow to prove that $A_n$ is normal in $S_n$? Note that $A_n$ is a group of even permutations on a set of length $n$. $S_n$ is the group of all permutations on $n$ symbols.

0
Q: Triple Integral in mvc

Reza HabibiHow i evaluate this $\iiint_{D}\sqrt{x^{2}+z^{2}}dxdydz$ where domain D is restricted by $y=x^{2}+z^{2}$ and plane $y=4$ ? I've already known that $y=x^{2}+z^{2}$ is paraboloid and but i'm not sure about the domain. Help me please!

Short title. Question contains please. Triple Integral in mvc
0
Q: $\sum_{\text{cyc}}\frac{1}{1+a}\le 1\implies{}a+b+c+2\le abc.$

Jack FrostI recently encountered an equality with the condition that $a,b,c$ are positive reals and $\sum\frac{1}{1+a}\le 1.$ The solution says that this condition is equivalent to $a+b+c+2\le abc.$ But I have a hard time figuring out why. Can anyone show why the two conditions are the same?

A title should not be all-MathJax; having some plain text helps with search and navigation. (from a bot)Normal Human 21 secs ago
0
Q: Integrate $\int\frac{x+1}{(x^2+7x-3)^3}dx$

MatusKHow can i solve something like that? $$\int\frac{x+1}{(x^2+7x-3)^3}dx$$ How should I start? Should I try rewrite it in partial fractions?

Short title. Short question. Integrate $\int\frac{x+1}{(x^2+7x-3)^3}dx$
0
Q: What is the chance of rolling a die and getting the number six three times at exactly 10 roles?

travis taI was asked this question in my statistic class. I thought the way to do this was (1/6)^3 x (5/6)^7, because that is getting six 3 times and not getting it 7 times. However, that's wrong, I figured that it's because that (1/6)^3 would be getting 6 three times in a row. Could you explain how to d...

Welcome to Math.SE, travis ta. This site uses MathJax formatting of formulas. More tips here. (from a bot)Normal Human 21 secs ago
0
Q: How can I find a value of $n$ which yields the minimal difference between $\dfrac{2^n}{88}$ and $\Big[\dfrac{2^n}{88}\Big]$?

barak manosLet $[x]$ denote the rounded (nearest integer) value of $x$. How can I find a value of $n$ which yields the minimal difference between $\dfrac{2^n}{88}$ and $\Big[\dfrac{2^n}{88}\Big]$? Empirically, I have noticed that the minimal value of $\dfrac{1}{11}$ can be achieved with any $n=5k+3$: $\...

Tall formulas in titles break the layout of question lists. Please replace \dfrac with \frac in the title. (from a bot)Normal Human 21 secs ago
0
Q: Quick question about Coset notation

Argon14I'm not looking for an answer to the actual problem, just what this notation could plausibly mean. Let $G$=$R$ modular addition $R$, and fix positive real numbers $a$ and $b$. Let $H$=<<$a$,$b$>>. Give a geometric interpretation of the cosets of $H$. What is that <>? I've never seen that notati...

Words such as question do not add information to titles. Please edit the title so that it better describes the specifics of your question. Do not hesitate to make it longer or include a formula if needed. More tips here. (autocomment)Normal Human 21 secs ago
0
Q: Use composition algorithm to generate random variates?

周庆特How to use composition algorithm to generate random variates whose cdf and pdf is as follows. CDF and PDF

 
9:01 AM
0
Q: How to show $\frac{\mathbb{Z}_m\times \mathbb{Z}_n}{\langle (a,b)\rangle}\simeq \mathbb{Z}_{\frac mc}\times \mathbb{Z}_{\frac nd}$?

Anjan3Please help me. Suppose we have the group $\frac{\mathbb{Z}_m\times \mathbb{Z}_n}{\langle (a,b)\rangle}$. We need to justify that (i) There exist $c, d$ such that $\langle (a,b)\rangle$ is isomorphic to the group $\mathbb{Z}_c\times \mathbb{Z}_d$ with $c|m, d|n$. (ii) $\frac{\mathbb{Z}_m\times ...

0
Q: How can I justify that the partial sums of (2n)^k/k! is less than the number (2n)^n / n!?

Lebron JamesI am currently using Rouche's Theorem from complex analysis but am working on an upper bound and want to show $$\sum_{k=0}^{n-1}\frac{(2n)^k}{k!}< \frac{(2n)^n}{n!}$$ Any suggestions are welcome. Thanks,

0
Q: What is the probability of drawing at least two yellow balls if you draw 4 times (without replacement)?

BNSlug14 balls total, 4 red, 4 blue, 3 green, 3 yellow. What is the probability of drawing at least 2 yellows? My first take on this is to calculate 1 - the chance of not drawing a yellow in 4 pulls. 1 - (11/14 * 10/13 * 9/12 * 8/11) However, I realized that this doesn't cover the case in that you c...

This site uses MathJax formatting of formulas. More tips here. (autocomment)Normal Human 20 secs ago
 
9:20 AM
0
Q: I have received rep points that I don't deserve

Rolf of SaxonyOn the following question:wxpython frame doesn't re-draw it's contents in while loop I added a bit of presentation to an existing answer and specifically said that my answer was not to be accepted. It was, how can I ensure that the person who posted the original answer gets their answer accepted,...

 
0
Q: Question regarding proof Erdos-Kac Theorem in Durrett's book.

Coroner_RexI have a question regarding the proof of Erdos-Kac central limit theorem in on p.117 of Probability: theory and example. 4th ed. Durrett used a probability model $$ S_n = \sum_{p\leq \alpha_n} X_p, $$ where he claims that $$ \frac{S_n - b_n}{a_n}, $$ where $ a_n^2 $ is the variance of the series ...

0
Q: convergence up to first transfinite ordinal number

fixedpointSuppose t₁,t₂,...,t_{k},... is a sequence from a compact metric space T. Consider a sequence of constrained optimization problems V(t₁) = max_{x∈X}{f(x):g(x,t₁)≥0} V(t₁,t₂) = max_{x∈X}{f(x):{g(x,t_{i})≥0}_{1≤i≤2}} .... V(t₁,t₂,...,t_{k}) = max_{x∈X}{f(x):{g(x,t_{i})≥0}_{1≤i≤k}}, where bot...

0
Q: Minimization of DFA

technoI have the following question I have minimized the DFA as the following since the states can only be partitioned to [S0][S1 S2]

Short title. Short question. Minimization of DFA
Welcome to Math.SE, fixedpoint. This site uses MathJax formatting of formulas. More tips here. (from a bot)Normal Human 26 secs ago
0
Q: The equation of a jelly bean curve?

KarlWhat is the equation of a curve with jelly bean shape? I have found a quartic equation for bean shaped curves, but nothing for jelly beans.

0
Q: Finding Ideals of a finite semigroup matchinary (GAP)

Babak S.As you see, I did a finite semigroup and then try to find its possible ideals in a very basic way (with the hand of a friend): > f:=FreeSemigroup("a","b");; a:=f.1;; b:=f.2;; s:=f/[[a^3,a],[b^2,a^2],[b*a,a*b^3]];; e:=Elements(s);; w1:=List([1..Size(e)],i->Elements(Semigroup...

Questions tend to get more attention when they have a tag for a broad area of mathematics relevant to the question. Some of these tags might fit. (from a bot)Normal Human 20 secs ago
0
Q: How to do rational expressions

user2355058Never was much of a math student but I am brushing up on my arithmetic and algebra for college. I am using sample questions from Accuplacer and then using video lectures and practice on Khan Academy. It is going well but sometimes the examples in the lecture are very trivial and I am currently ...

Welcome to Math.SE, user2355058. This site uses MathJax formatting of formulas. More tips here. (autocomment)Normal Human 20 secs ago
0
Q: How to possible comnations for brute force search

Mr.VNI have $12$ players which one player can choose an arbitrary number in the set $\mathcal{A}=\{1,2,3 \}$. I get stuck in how to define and display all possible combinations by using Matlab. Do you have any idea for this?

 
9:36 AM
0
Q: Should it be possible to move a comment from an answer to the OP?

KjartanI've come across these situations while reviewing occasionally. As an example, see this answer *(assuming it has not been deleted by the time you read this). The answer here should clearly be a comment on the original question, so I would recommend it be deleted. There may however be several us...

0
Q: MVC View not deisplaying returned data from controller

mojiI am sending data through HttpClient (Json) from console app to web app, everything is working well but the only issue Iam facing that data sent from controller to view is not displayed. I debugged the program and i can see that data are returned to the view. Following are the codes: Constroller...

 
0
Q: Confusion on Theorem in Kato's book

PeterOn page 432 (pdf-page: 455) of Kato's book perturbation theory of linear operators, I do not understand why in Theorem 1.15 $$H_n = \int dE_n(\lambda)$$ instead of the ususal thing $$H_n=\int \lambda dE_n(\lambda).$$ Is this a misprint?

Title contains confus. Confusion on Theorem in Kato's book
 
9:54 AM
0
Q: Method of lines for elliptical PDEs (Laplace Heat Equation)

DoubleOsevenConsider the following steady state problem $$\Delta T = 0,\,\,\,\, (x,y) \in \Omega, \space \space 0 \leq x \leq 4 ,\space \space \space\space 0 \leq y \leq 2 $$ $$ T(0,y) = 300, \space \space T(4,y) = 600$$ $$ \frac{\partial T}{\partial y}(x,0) = 0, \space \space \frac{\partial T}{\partial y...

Tagged differential-equations but mentions "partial". Method of lines for elliptical PDEs (Laplace Heat Equation)
 
0
Q: Is Golden badge required to close a question as duplicate?

StarkeenCan I close a question as duplicate if I have 20k reps but I don't have a golden badge on any of the question related tags?

1
Q: Is this major edit justified?

Ian KempI just made a very major edit to an answer - current version (my edit) | previous revision. My rationale for editing, instead of posting a separate answer, is that the code/plugin in question is that same and that the previous version of the answer, among other things: was extremely verbose, co...

 
0
Q: Stochastic(Markov) matrix

user262860We say that a matrix $P $ = $(P_{ij} : i, j \in I)$ where $I$ is a countable set, is stochastic if every row $(P_{ij} : j \in I)$ is a distribution. What do we mean by $distribution$ here?

 
0
Q: Delete old tags which are not used

demonplusWe have some tags with 0 questions, 0 followers, (sometimes also no description, probably this is not so important) and rather old. Probably if such a tag is more than some time old (say, 1 year) it can be safely automatically deleted?

 
0
Q: Manifold and maximal atlas

MSE1) I didn't understand really what is a maximal atlas. Is it as set of compatible chart maximal in the sens that adding one more chart will yield the atlas not compatible ? 2) Let two atlas $\mathcal A$ and $\mathcal A'$. So if they are compatible, they are both in a maximal atlas $\hat{\mathcal...

0
Q: Why such graph exists?

lisuwu12Let V=[2015]. Show that there is a graph G=(V, E) such that for any A, B $\subseteq$ V where A $\cap$ B= $\emptyset$ and |A $\cup$ B|=5, there exists $x \in V-(A\cup B)$ such that x is adjacent to all vertices in A but none vertices in B.

 
10:27 AM
0
Q: When is implication true?

user216094If we have $p\implies q$, then the only case the logical value of this implication is false is when $p$ is true, but $q$ false. So suppose I have a broken soda machine - it will never give me any can of coke, no matter if I insert some coins in it or not. Let $p$ be 'I insert a coin', and $q$ -...

0
Q: Who can help me proof the theorem about RSK correspondence

Yi ZhangI'm studying the RSK correspondence,but I can't proof the following theorem.Can anyone help me? enter image description here

Welcome to Math.SE, Yi Zhang. Words such as help do not add information to titles. Please edit the title so that it better describes the specifics of your question. Do not hesitate to make it longer or include a formula if needed. More tips here. (autocomment)Normal Human 34 secs ago
0
Q: Integrating functions

Sat DIntegrate $f(x)$ from 0 to 1 where $f(x) = \frac{x^3-1}{lnx}$ I received this problem and a variety of others in an advanced mathematics exam. I tried a classical trigonometric substituition approach with $x=sec^\frac{2}{3}t$ but it gets really long and still gives no answer. I tried multiplying...

0
Q: What are the applications of functional analysis?

chiI recently had a course on functional analysis. I was thinking of studying the applications of functional analysis. I came to know it had some applications on calculus of variations. Can anyone give a brief on what are the applications of functional analysis? Also, please suggest some good book...

0
Q: Linearization of PDE: $0$ is an eigenvalue since all translates of travelling waves are also travelling waves

M. MeyerConsider the following PDE: $$ u_t=u_{xx}+f(u)-w,~~~~~w_t=\varepsilon (u-\gamma w),~~~~~~~~~(1) $$ where $f(u)=u(u-a)(1-u), 0<a<\frac{1}{2}, \varepsilon,\gamma >0, \varepsilon\ll 1,\gamma\ll 1$. A travelling wave for (1) is a solution that is a function of the single variable $\xi=x-ct$, i.e., $...

0
Q: Diameter of certain graphs?

M.BadaouiI guess the following is true but i do not know how to prove it: Let $(G_i, i\in \mathbb{N}^+)$ be family of connected $d$-reuglar graph (same $d$ for all $i$), such that $v(G_i)\rightarrow \infty$ as $i \rightarrow \infty$. Then diameter$(G_i)\rightarrow \infty$ as $i\rightarrow \in...

0
Q: Math backbone in biology/genetic

dshulginAre there any good books, that shows some advanced math models in genetics particularly? The problem is i don't see any good math model in genetics interactions exept combinatorics. I'd like to know what else math structures and rules are there in genetics. I'd like create strong math backbone f...

Welcome to Math.SE, dshulgin. Please don't use (self-learning) tag just because you were self-studying. This tag is only for questions about the process of self-studying. (autocomment)Normal Human 1 min ago
0
Q: Problem on Borel real line

Anoop BhushanWe are given a distribution with mean u, and variance sigma square. An unknown random variable X exists whose values take on belong to a range of numbers. This range of numbers from which X takes values depends on an integer k. We define P{X(w):w€(some range on the borel line)} = p For what what ...

 
10:53 AM
0
Q: Are [causuality] and [causal-inference] tags the same?

TimThere are two tags causality and causal-inference that seem identical or at least closely related -- aren't they? Reviewing the posts tagged by both tags shows that there is not much difference between them. Maybe they should be merged?

1
Q: What shall I do when I see spam (in one of the review queues)?

Martin - マーチンIn a few occasions if you use the review queues, you will find a post that is spam. The definition of spam according to StackExchange (and the internet in general; source) is: What makes something spam and when should I flag it? A post should be marked as spam ONLY when it contains an unsol...

 
Short title. Title contains problem. Problem on Borel real line
0
Q: Proving a contraction mapping

ArthurI have difficulties understanding the proof of a contraction mapping in context of ordinary differential equations. For example: We have a banach space $(Y,||\cdot ||)$, where $||y||=\sup\{\frac{|y(x)|}{x}:0<x\leq 1\}<\infty$ Also, we have the map: $T:Y\rightarrow Y$, where $Y$ is the set of ...

0
Q: Proof inequation smaller than one

TonioHello. How can I show that $$\sum_{k = 0}^{\infty} \frac{ \log(t)^k}{k!}h^k ≤1 $$ is right?

0
Q: Conditional Expectations: Minimal Square error???

Fabian WernerThe problem I'm fighting with right now originally comes from signal noise detection: Given a prob. space $(\Omega, \mathcal{A}, \mu)$ and random variables $X, Y : \Omega \to \mathbb{R}$ (where $X$ encaptures a 'true' signal and $Y$ contains its noisy variant $Y = r(X) + \text{error}$ that was ob...

0
Q: Draw balls in the urn Probabilities

user290335can you help me on this? An urn contains 7 balls, 4 of which are black and the rest white. 1) Starting with all 7 balls in the urn, balls are drawn randomly with replacement. Find the probability that among the first 5 balls which are drawn a) 2 of them are white. For 1 a) I used Binomial...

This site uses MathJax formatting of formulas. More tips here. (autocomment)Normal Human 30 secs ago
0
Q: About Lebesgue-Radon-Nikodym Theorem

Li JingyangLebesgue-Radon-Nikodym Theorem shows that: If $\mathbb{M}$ is a $\sigma$-algbra on the set X. $\mu,\lambda$ are a $\sigma$-finite positive measure and $\sigma$-finite signed measure on $\mathbb{M}$ respectively. then $\lambda$ has a decomposition. I wander why we do not consider the case when $...

0
Q: An example of totally ordered field with cardinality larger than continuum.

ᴊ ᴀ s ᴏ ɴIs there any totally ordered field with cardinality larger than the continuum? If such field exist, please give an example (an simple one if possible).

0
Q: simplex algorithm 0

Olya12simplex algorithm Attaching coves $10$ reinforcement wire length $50$ m and $30$ m. The armature must have a size of $8$ m and $6$ m and $8$-m two times more than $6$-m cove cut so as to obtain the largest number of sets of armatures I can not understand how to do it

Short title. Title ends with a digit. simplex algorithm 0
0
Q: operations with sets

p.mcproof or disprove : a) A ∩ (B \ C) = (A ∩ B) \ (A ∩ C) b) A ∩ (B \ C) = (A ∩ B) \ C c) A \ (B ∩ C) = (A \ B) ∪ (A \ C) d) (A \ B) ∪ (B \ C) ∪ (C \ A) = (A ∪ B ∪ C) \ (A ∩ B ∩ C) can someone help this please?

Short title. Question contains please. operations with sets
0
Q: For every k Part of the natural numbers

allFor every k Part of the natural numbers show that we have n So that: 2^n+3^n-1,2^n+3^n-2,...,2^n+3^n-k Not all prime numbers

This site uses MathJax formatting of formulas. More tips here. (autocomment)Normal Human 33 secs ago
 
11:25 AM
0
Q: Spam becoming frequent?

Star OSToday, seeing AskUbuntu, it seems that spam is getting frequent, 30 minutes a spam thread is being posted. The most recent one was about "making people to look younger". The first one i saw today was about a game..

 
11:37 AM
0
Q: Jacobi Symbol Uniqueness

IgorI know that Legendre Symbol is unique non-trivial group homomorphism from $U(Z_n)$ to ${-1,1}$. Is the same true for Jacobi symbol? I can prove it for the case when $n$ is power of odd prime. In this case the kernel of non-trivial homomorphism must be subgroup of size $\frac{\phi(n)}{2}$ and $U...

0
Q: Find eigenvalue of this integral

GahirShow that the number of $\lambda$, eigenvalue of $$\int_a^b \int_a^b K(s,t)^2 ds dt $$ is finite.

0
Q: Question about periodization of the Dirichlet Kernel

NimmerThe Dirichlet Kernel on the real numbers is: $\mathcal{D}_\mathbb{R}(x) = \frac{sin(2 \pi R x)}{\pi x}$. Let $D^*_N(x) = \sum_{|n| \leq N - 1} e^{2 \pi i nx} + \frac{1}{2}(e^{-2 \pi iNx} + e^{2 \pi iNx})$ denote the Dirichlet kernel for functions of period 1. Show that $\sum_{n = -\infty}^{\inf...

Words such as question are uninformative in titles. Please edit the title so that it better describes the specifics of your question. Do not hesitate to make it longer or include a formula if needed. More tips here. (from a bot)Normal Human 27 secs ago
 
12:07 PM
0
Q: Prove that $m-n$ divides $p$

User 1upon0Let $a_1,a_2,a_3$ be a non constant arithmetic Progression of integers with common difference $p$ and $b_1,b_2,b_3$ be a geometric Progression with common ratio $r$. Consider $3$ polynomials $P_1(x)=x^2+a_1x+b_1, P_2(x)=x^2+a_2x+b_2,P_3(x)=x^2+a_3x+b_3$. Suppose there exist integers $m$ and $n$ s...

0
Q: Winding number of a closed Archimedean spiral

狼狼狼Why the winding number in the image is different for these different points? Should not it be +3 for all? I really don't understand this... http://s8.postimg.org/sda6gvdh1/windingnumberclosedarch.png

0
Q: Transportation problem with complication

Ivan AfonichkinI have a transportation problem with complication. Essense of complication -- shipping is carried out with a trucks. Each truck has limited capacity. Each truck has the same capacity. Let's call capacity of eack truck -- K. How to reduce this problem to the linear programming problem?

0
Q: Is the proof of the statement make sense?

chuacklPlease refer this link for some background material http://www.docdroid.net/161p6/curve.pdf.html So i propose a statement to a online tutor, the answer at the below link is the proof of the statement? I couldn't get it. http://postimg.org/image/j26erz5k3/

0
Q: Proof of $\lim_{x\to\infty}\frac{\text{Ei(x)}}{e^x}=0$

dielectricI encountered the following limit while doing calculation $$\lim_{x\to\infty}\frac{\text{Ei(x)}}{e^x}=0$$ which is equivalent to $$\lim_{x \to \infty }e^{-x}\sum_{n=1}^{\infty}\frac{x^n}{n·n!}=0$$ and $$\lim_{x\to\infty}\int_0^\infty \frac{e^{-t}}{t-x}dt=0$$ where the integral is undestood as the...

0
Q: Minkowski sum: open/closed?

k.dkhkIf a set A is open, and you take the Minkowski sum with any other set B (open or closed), will A+B then be open? In my head it makes sense if it is open but I can't prove it on paper.

Short title. Short question. Minkowski sum: open/closed?
0
Q: Rope wrapped around pole Friction

RSparkesA rope is wrapped $M$ whole turns round a cylindrical post, the two ends of the rope going in opposite directions. The coefficient of friction between rope and post is $0.25$. It is desired that by pulling with a force of $1N$ on one end of the rope, I can prevent the rope from moving away from m...

Questions tend to get more attention when they have a tag for a broad area of mathematics relevant to the question. Some of these tags might fit. (from a bot)Normal Human 20 secs ago
 
-2
Q: Is my grammar not clear enough?

dananjayaI tried to post this as a question on Stack Overflow: I am developing a web application using java.in it i want to send a text message to a mobile.please help me with some example. but got the following message, and was not allowed to submit: This question body does not meet our quality...

 
0
Q: $f(x+y)+f(xy)=f(x)+f(y)+f(x)f(y)$

Mohsen ShahriariI was reviewing this question and got motivated to solve this general problem: Find all functions $f:\mathbb R\to\mathbb R$ such that for all real numbers $x$ and $y$, $$f(x+y)-f(x)-f(y)=\alpha(f(xy)-f(x)f(y))\tag0$$ where $\alpha$ is a nonzero real constant. I found out that it can be ...

A title should not be all-MathJax; having some plain text helps with search and navigation. (autocomment)Normal Human 20 secs ago
0
Q: solving sets with help

p.mcThe symmetric difference can be expressed as the union of the two sets, minus their intersection- how to prove that ?

Words such as help do not add information to titles. Please edit the title so that it better describes the specifics of your question. Do not hesitate to make it longer or include a formula if needed. More tips here. (autocomment)Normal Human 20 secs ago
0
Q: Finite integral domain

Liu ShengxuanI encountered a problem: Every finite integral domain is isomorphic to $ \mathbb{ Z }_{p} $. I know that finite integral domain is isomorphic to a field, but I have no idea on how to construct a homomorphism to $\mathbb{Z}_{p}$ (or maybe it is wrong, but I haven come up with a conterexample).

0
Q: Reverse cross product

Hacker BoyGood afternoon, I am currently having issues solving a problem: Let A and B have position vectors a = 2i – j + 3k and b = 5i – 2j + k respectively. The force F = 3i – 5j – k passes through the point A. Show that the vector moment of force, F about the point B is equal to 3(3i + j + 4k...

 
12:42 PM
0
Q: Voting to close a question as off-topic because it's a notorious open problem

WolfgangI just voted to close a question which was equivalent to Goldbach's conjecture. When I had to give the reason, I didn't really know what to pick. Off-topic? It is definitely a research question, so... Duplicate? Well, hopefully not, at least not exactly. When it was closed some hours later, I fou...

 
0
Q: Minkowski sum: open/closed?

k.dkhkIf two sets (A,B) in R^n are closed. Will A+B then be closed or open? In my head it makes sense if it is closed but I can't prove it on paper.

Short title. Short question. Minkowski sum: open/closed?
0
Q: if $c>1$ then $\sup\limits_{0 < r < 1}$ is finite

soyoif $c>1$ then $\sup\limits_{0 < r < 1}\left(\frac{1}{2\pi}\int_0^{2\pi}|\dfrac{1}{1-re^{it}}‎ ‎\left( ‎\dfrac{1}{re^{it}}‎ \log \dfrac{1}{1-re^{it}} ‎\right)‎^{-c}|~ dt \right) ~<~ \infty$‎

 
0
Q: Can you just point someone in the right direction and how vague are you allowed to be with this?

Paranoid PandaI recently asked this question, and this was one of the answers I got: I'm running Ubuntu 14.04.03, so YMMV. Investigate some of these packages: $ apt-cache search ssh | grep multiple byobu - powerful, text based window manager and shell multiplexer vcsh - Version Control System for $HOM...

 
1:00 PM
0
Q: how to solve fokker-planck equation using space-time laplace transform?

nitinI was wondering about how to solve a simple linear Fokker-Planck equation using space-time laplace transform on space interval $(0 \ \infty)$. $\frac{\partial f(x,t)}{\partial t}= k_1 \frac{\partial f(x,t)}{\partial x} + k_2 \frac{\partial^2 f(x,t)}{\partial x^2}$. The usual method is to do lap...

Tagged pde, differential-equations. Tagged differential-equations but mentions "partial". how to solve fokker-planck equation using space-time laplace transform?
 
0
Q: New site design menu memory

DavidTFirst meta post so be kind if I have posted in the wrong area. Im liking the updates to the site design except for one thing. Previously I am sure that I could select some options from the menus for filtering questions. Namely, Unanswered -> newest (tab) I could then answer/view a question and ...

 
0
Q: Finding simple worked exercises for Category Theory

Jerome ShahI am in the process of learning Category Theory with the purpose of being able to create a game that will help explain it to others in a simple way. I have read many texts and articles about it. While I have learnt some of the definitions, I have found it hard to be able to create concrete exampl...

0
Q: i have a few questions regarding differentiation using the first principle

JOE KABIRUDifferentiate the following using the first principle f(x)=x^3/2 f(x)=1/square root of x i have tried the questions but i am stuck in the second step

Questions tend to get more attention when they have a tag for a broad area of mathematics relevant to the question. Some of these tags might fit. (from a bot)Normal Human 21 secs ago
0
Q: Projecting a surface

Mirac7I’m designing a 3d game and am having trouble computing where the mouse pointer is actually pointing on the flat map when taking into account perspective which skews the view. Let’s assume that the flat plane is part of the map, and the tilted plane is the viewport. Player’s camera is rotated ...

Short title. Projecting a surface
 
-1
Q: How can we check that the question is marked as Bounty?

NehalI just want to know that, how the users get notify that any questions is marked/awarded as Bounty. So that whoever answers that question receives the bounty points which effects their reputation. EDIT : I just want to know that, if a question is marked as bounty, how do other users get to know ...

 
1:18 PM
0
Q: Sum of Euler totient function

kertmeyenkeleWe define a new function: Psum(N)=Sum[1<=a<=N, 1<=b<=N, phi(a*b)] For example, PSum(2)=phi(1*1)+phi(1*2)+phi(2*1)+phi(2*2)=5 You are given that PSum(10)=1271, PSum(100)=10813139, and PSum(1000)=107260466665 Find PSum(10000000) Give the rightmost 9 digits as your answer I can find the small val...

Welcome to Math.SE, kertmeyenkele. Questions tend to get more attention when they have a tag for a broad area of mathematics relevant to the question. Some of these tags might fit. (from a bot)Normal Human 21 secs ago
0
Q: Delta Method corollary

STFConsider the Delta Method as stated in van der Vaart Theorem 3.1 at page 26 (you can find the page here https://books.google.co.uk/books?id=UEuQEM5RjWgC&pg=PA32&lpg=PA32&dq=van+der+vaart+theorem+3.1+because+the+sequence+converges+in+distribution&source=bl&ots=mnRJLD8XLC&sig=inIMmSPvWDfrPc6r4U7dnu...

0
Q: Anyone knows how to solve a linear EQUATION SYSTEM CONTAINING SUMMATION with Maxima/Maple/Mathematica?

G.CThis problem has been extreme confusing for me since 2 weeks ago. I've tried many times with Maxima but still i got no results. Following is the equation system: equations with related functions. To be computed are b1[n], b2[n], b3[n], a4[k] and a5[k]; a3[k] is already obtained before. R1,R2,R...

0
Q: Finite group G is abelian if G has an automorphism $\sigma$ such that $\sigma(g) = g \iff g = 1$ and $\sigma^{2}=id$

taroProve that a finite group G is abelian if G has an automorphism $\sigma$ such that $\sigma(g) = g \iff g = 1$ and $\sigma^{2}(x)=x$ for all $x \in G$. Show that every element of G can be written in the form $x^{-1}\sigma(x)$ and apply $\sigma$ to such an expression.

Please don't use (self-learning) tag just because you were self-studying. This tag is only for questions about the process of self-studying. (autocomment)Normal Human 21 secs ago
0
Q: Error approximation between

behmor67We have approximation of $f(x)=\sqrt(x)$ such $P(x)$ in interval $[a,b]$ . How we can approximate the accuracy of my approximation in this interval?

Short title. Short question. Error approximation between
 
1:39 PM
0
Q: Are software comparisons allowed and if so how broad are you allowed to be?

Paranoid PandaAre software comparisons considered on-topic here? I mean Ubuntu software of course, and probably software which is specific to it or at least Linux but runs on Ubuntu. And how broad is considered 'too broad' in this case? I mean, can you just ask what is the difference and why you would use one ...

0
Q: What is the difference between Cross Validated and Math SE for homework?

BCLCWhy is it that CV requires homework or the like to be tagged w/ self-study, but Math SE did away w/ homework tag a long time ago? Why don't the arguments for no more homework tag in Math SE apply to CV? Why not treat all questions on CV the way self-study questions are treated?

 
0
Q: How to solve a set of 3 nonlinear equations (lines in space)

jRambI would like to calculate the following: There are 3 points in space of which the coordinates are known: C(a,b,c) M(d,e,f) A(g,h,i) Each point is a starting point of a line with a certain length. These lengths are known: |CB|=l1 |MB|=l2 |AB|=l3 There exist 2 possible points (B1 and B2) ...

Welcome to Math.SE, jRamb. This site uses MathJax formatting of formulas. More tips here. (from a bot)Normal Human 21 secs ago
0
Q: Language of all the binary words that not contains continuum of more then $3$ zeros

NehoraiI need to write a regular expertion for the language of all the binary words that not contains continuum of more then $3$ zeros, for example $0011110100\in L,\,\,\,\,\,\,\,\,\,11000001100\notin L $ My try: $[(0+1)^*(00+1^*)^*(000+1^*)]^*$ My attempt is correct?

0
Q: How to write a rigorous proof for Normalisers $N_{G}(H)$ being the largest subgourp of $G$ such that $H \unlhd N_{G}(H)$

Rescy_ Prove that $N_G(H)=\{g \in G| gHg^{-1}=H\}$ is the largest subgroup of $G$ such that $H \unlhd N_G(H)$. I have an idea of the proof that, if we assume $S \leq G$ with $H \unlhd S$ then $$\forall s \in S, \ sHs^{-1}=H$$ We know that for $S$ to be the largest subgroup of $G$ with this propert...

0
Q: permutations, product rule

GurungLet n>=2 be an integer and consider two fi xed integers a and b with 1<=a<=b<=n.  Use the Product Rule to determine the number of permutations of {1, 2,...,n} in which a is to the left of b.  Consider a uniformly random permutation of the set {1,2,....,n}, and de note the event A = "in this pe...

 
0
Q: What is unclear about my question on probability distributions?

BCLCDoes it make sense to have a joint pdf or joint cdf of an infinite collection of random variables? was closed for being unclear. Ummmmm, essentially I was asking if we can extend the notion of joint pdf or joint cdf to an infinite collection of random variables. What is wrong w/ my question, and...

 
2:00 PM
0
Q: The sum of two strongly convex functions

Chencheng LiAssume that f is a-strongly convex and g is b-strongly convex, then I want to know that f+g is ?-strongly convex?

0
Q: Solve $3y + 2 \equiv 3\ (5)$ in math exercise

Double H AttackI'm stuck at the ending part of a math exercise on congruences. I must solve the following system of congruences $S$: $x \equiv 2\ (3)$ $x \equiv 3\ (5)$ I was first asked to give the remainders of the division of $3y +2$ by 5, with knowing the remainders of the division of $y$ by 5. Here's ...

0
Q: Matrix Integral Problem, what should I do

mysteryjoeA,B squares exp(t(A+B))=exp(tA)+$\int_0^t exp((t-s)A)*B*exp(s(A+B))*ds$ I found it from problem sets. It seems to difine a new kind of matrix operation. I can check its truth when A,B are scalars (n=1), but fail to prove it for general cases. The matrices do not commute and I cannot think of ...

0
Q: Semaphore in java.What is it exactly.

ShubhamWhat is exactly semaphore in java?Can someone explain it in simple words by citing a short example.

0
Q: convex polytopes

jdizzlefor arbitrary dimension, what are the convex polytopes such that all vertices share a facet of some dimension, which is not the top facet (the entire polytope), with all other vertices? One example is simplexes of any dimension, as all vertices share an edge with all other vertices. Are there oth...

Short title. convex polytopes
0
Q: In rhombus ABCD triangle AOB is congruence to BOC?

narendarIs Triangle AOB congruence to Triangle BOC?

0
Q: Number of permutations for which value i is the leftmost element of an increasing subsequence

John WatsonFor any i where 2 <= i <= n, use the product rule to determine how many permutations of the set {1,2,…,n} are there such that i is the leftmost value in an increasing subsequence.

0
Q: Inequality + Percentage

user256670"In a class, 40% of students want to study physics, while less than 33 1/3% of them want to study Chinese History. If the number of students who want to study physics is 3 more than that of students who want to study Chinese History, at most how many students are there in the class?" Don't know...

Short title. Question contains please. Inequality + Percentage
0
Q: Expanding $f\left ( z \right )=\frac{e^{az}}{1+e^{z}}$ about $z= i\pi$

Shemafied$$f\left ( z \right )=\frac{e^{az}}{1+e^{z}} ,\left ( a\in\left ( 0,1 \right ) \right )$$ The point $z=i\pi$ is one of the nonremovable singularities of this function. In order to expand it about that point I introduced the change of variables $z=\omega +i\pi$ and expanded the function $f\left(\o...

Questions tend to get more attention when they have a tag for a broad area of mathematics relevant to the question. Some of these tags might fit. (autocomment)Normal Human 21 secs ago
 
2:16 PM
0
Q: reward not solved question

BernardI had a question connect two subnets Linux which is not solved yet. But user TomS has done many constructive attempts to solve. Which does give me a direction were to search further. I would like reward him for that. How can I do so, because the question is not solved.

 
0
Q: Addition two Ideals I+J of polynomials

user165210i tried to connect two ideals: I = {x^3 + 1} J = {x^3 - 1} so I + J = ? i tried like this: I + J = = = ... ? tnx

This site uses MathJax formatting of formulas. More tips here. (autocomment)Normal Human 21 secs ago
0
Q: Why die Euclid call 6 a perfect number?

K.B.The old Greek did not consider 1 a number. Nevertheless Euclid called 6 = 1+2+3 a perfect number. How could he use 1 which was not a number?

0
Q: Solve following sum

user164612enter image description here I am very unsure how to do this, As the sign changes I would seperate them in two series, one positive and one negative - 1/((2n-1)(2)^(2n-1)) and -1/((2n)(2)^(2n)) However I can't evaluate them right.

Short title. Solve following sum
 
-3
Q: What happen to Did's account?

An old man in the sea.I've just seen that Did's account has been suspended. What happened to him/her? How can someone with that much reputation lose their account?

 
0
Q: 1/p in p-adic number system?

awvtto expand $1/p$, I tried first letting $1/p = a+b*p+c*p^2+d*p^3+...$ and it is $1=a*p+b*p^2+...$ but I guess there's no way to make the equality hold. it's somewhat similar to dividing by 0. is it possible? I'm new to p-adic theory, everything looks just confusing.

0
Q: Problem from functional analysis that I have no idea where to start

uptowndowndownI have no idea where to start or what kind of concept I should use for the following problem: Suppose $f \in L^1(\mathbb{R})$. Show for almost all $x \in \mathbb{R} $ we have $$\lim_{h\to 0} \frac{1}{h} \int_0^h |f(x+h)-f(x)| dt = 0$$ Could someone help me with this problem? Thanks!

0
Q: Game Theory poker game

Jagger Fungsa4. Consider the following simplified version of a game of poker. There are two players and three cards in a deck: Jack, Queen and King of spades. Player 1 always gets a Queen. Player 2 gets a Jack with probability 0.5 or a King with probability 0.5. In the first round, player 2 can either Raise o...

 
2:34 PM
0
Q: Order of displaying answers for a question

ravindraHave a look at below question : How to check if two words are anagrams I don't know the criteria for showing order of answers but you can see that lot of negative score answers are hiding good answers. I think that -ve score answers should be moved down and +ve score answers should move up. ...

 
0
Q: Sequence of cts functions converging to a non cts function. Show the degree of sequence functions goes to infinity.

NateIf f ∈ C[a,b] is not a polynomial, then show that for any sequence of polynomials {p_n} that converges to f uniformly, one must have that m_n = degree of p_n → ∞.

0
Q: Proof verification: if $E$ is a finite dimensional subspace of a normed space $X$, then $E$ is a closed subspace.

I'm mostly just an idiotCould someone please verify the following proof for this statement: If $E$ is a finite dimensional subspace of a normed space $X$, then $E$ is a closed subspace. Proof: If $E$ is a finite dimensional subspace of normed space $X$, then $E$ is complete. Let $\{x_n\}_{n=1}^\infty$ be a Cau...

0
Q: Bayesian game theory

captaincookJohn Harsayni showed that a mixed strategy equilibrium of a perfect information game can be thought of as an approximation to an equilibrium (i.e. Bayesian NE) of a game where each player has a slight amount of incomplete information about the exact preferences of the other players. Consider the ...

Short title. Bayesian game theory
0
Q: Simplex algorithm reaches optimal solution but with negative slack variables

JoeI am working on a VBA algorithm that will solve simple versions (single stock length, <1000 patterns) of the Cutting Stock problem, and after a lot of research I have managed to write a VBA program that correctly calculates the patterns available. As the Cutting Stock problem is a minimisation p...

Welcome to Math.SE, Joe. Questions tend to get more attention when they have a tag for a broad area of mathematics relevant to the question. Some of these tags might fit. (from a bot)Normal Human 21 secs ago
0
Q: Log Test for Integrals

redmageLet $f$ be locally integrable and positive on [0,$+\infty$) such that $L \;:= \;lim_{x\rightarrow \infty} \dfrac{-ln f(x)}{ln x}\;$ exists in $\bar{\mathbb{R}}$. Prove that $\int^\infty_0 f$ converges if $L > 1$ and diverges if $L < 1$. The idea seems close to the Root Test for Integrals, but I...

0
Q: Complex filter factorizations

mathreadlerThere is a famous low pass filter $[1,2,1]$ in signal processing which can be factored in the sense of a convolution product : $[1,1] * [1,1]$. Today when I was bored, I investigated some complex valued filters and came to the conclusion that for $k = 3$ and $k = 5$ and $z_0 = e^{2\pi i/k}$, i.e....

 
00:00 - 15:0015:00 - 00:00

« first day (30 days earlier)      last day (544 days later) »